Difference between revisions of "2000 AMC 10 Problems/Problem 4"

(New page: Let <math>x</math> be the fixed fee, and <math>y</math> be the amount she pays for the minutes she used in the first month. <math>x+y=12.48</math> <math>x+2y=17.54</math>. <math>y=5.06</...)
 
(Problem)
 
(13 intermediate revisions by 5 users not shown)
Line 1: Line 1:
 +
==Problem==
 +
 +
Chandra pays an on-line service provider a fixed monthly fee plus an hourly charge for connect time. Her December bill was <math>\$12.48</math>, but in January her bill was <math>\$17.54</math> because she used twice as much connect time as in December. What is the fixed monthly fee?
 +
 +
<math>\textbf{(A)}\ \$ 2.53 \qquad\textbf{(B)}\ \$ 5.06 \qquad\textbf{(C)}\ \$ 6.24 \qquad\textbf{(D)}\ \$ 7.42 \qquad\textbf{(E)}\ \$ 8.77</math>
 +
 +
==Solution==
 +
 
Let <math>x</math> be the fixed fee, and <math>y</math> be the amount she pays for the minutes she used in the first month.
 
Let <math>x</math> be the fixed fee, and <math>y</math> be the amount she pays for the minutes she used in the first month.
  
 
<math>x+y=12.48</math>
 
<math>x+y=12.48</math>
<math>x+2y=17.54</math>.
+
 
 +
<math>x+2y=17.54</math>
  
 
<math>y=5.06</math>
 
<math>y=5.06</math>
<math>x=7.42</math>.
+
 
 +
<math>x=7.42</math>
 +
 
 +
We want the fixed fee, which is <math>\boxed{\text{D}}</math>
 +
 
 +
==See Also==
 +
 
 +
{{AMC10 box|year=2000|num-b=3|num-a=5}}
 +
{{MAA Notice}}

Latest revision as of 09:49, 8 November 2021

Problem

Chandra pays an on-line service provider a fixed monthly fee plus an hourly charge for connect time. Her December bill was $$12.48$, but in January her bill was $$17.54$ because she used twice as much connect time as in December. What is the fixed monthly fee?

$\textbf{(A)}\ $ 2.53 \qquad\textbf{(B)}\ $ 5.06 \qquad\textbf{(C)}\ $ 6.24 \qquad\textbf{(D)}\ $ 7.42 \qquad\textbf{(E)}\ $ 8.77$

Solution

Let $x$ be the fixed fee, and $y$ be the amount she pays for the minutes she used in the first month.

$x+y=12.48$

$x+2y=17.54$

$y=5.06$

$x=7.42$

We want the fixed fee, which is $\boxed{\text{D}}$

See Also

2000 AMC 10 (ProblemsAnswer KeyResources)
Preceded by
Problem 3
Followed by
Problem 5
1 2 3 4 5 6 7 8 9 10 11 12 13 14 15 16 17 18 19 20 21 22 23 24 25
All AMC 10 Problems and Solutions

The problems on this page are copyrighted by the Mathematical Association of America's American Mathematics Competitions. AMC logo.png